微積5.5.6

前に戻る トップへ戻る

問題5.5.6

次の広義の重積分の値を求めよ。

(1)$\displaystyle \iint_D \dfrac{xy}{(x^2+y^2)^3}dxdy$ $D:\{(x,y)|\ x,y,\geqq 1\}$

(2)$\displaystyle \iint_D x^2 e^{-(x^2+y^2)}dxdy$ $D:\{(x,y)|\ x,y,\geqq 0\}$

 

《ポイント》

教科書の定理5.5.5~5.5.7を利用します。積分値を定義するために以下の3条件を満たす有界閉領域を考えます。

(ⅰ)$D_1 \subset D_2 \subset D_3 \subset \cdots \subset D$
(ⅱ)$\displaystyle \bigcup^{\infty}_{n=1}D_n=D$
(ⅲ)$D$に含まれる有界閉領域は、いずれかの$D_n$に含まれる

なお、本問では扱っていませんが、2重積分における広義積分は特異点をもつ有界閉領域でも定義できる場合があります(本問は有界閉領域の列を有界でない領域に近付けるタイプの重積分です)。

とはいうものの、実用上はこのような込み入った議論をすっ飛ばしても値は求められます。結局は有界な閉領域の境界をどんどん広げるだけの操作なので、広義積分の2次元版だと考えても差し支えありません。特に本問の領域の境界線は座標軸に平行なので長方形領域を無限遠まで広げればOKです。以下の解答例ではより直感的な極限のとり方で積分値を求めてみます。ここでは新しく定義した領域$D’$が、必要に応じて上記の3要件を満たすように構成できる、ということを自明とします。

(2)では定理5.5.7を利用します。

 


 

《解答例》

(1)$\displaystyle \iint_D \dfrac{xy}{(x^2+y^2)^3}dxdy$ $D:\{(x,y)|\ x,y,\geqq 1\}$

$D’$を$$D’=\left\{(x,y) \middle|\ 1 \leqq x \leqq a,1 \leqq y \leqq b \right\}$$と定めると、$$\iint_D \dfrac{xy}{(x^2+y^2)^3}dxdy=\lim_{\begin{align} a \to \infty \\ b \to \infty \end{align}}\iint_{D’} \dfrac{xy}{(x^2+y^2)^3}dxdy$$が成り立つ。

$$\begin{align} & \displaystyle \iint_{D’} \dfrac{xy}{(x^2+y^2)^3}dxdy \\
=&\int^{a}_{1}dx \int^{b}_{1} \dfrac{xy}{(x^2+y^2)^3}dy \\
=&\int^{a}_{1}\left[-\dfrac{1}{2}\cdot\dfrac{1}{2}\cdot\dfrac{x}{(x^2+y^2)^2}\right]^{y=b}_{y=1}dx \\
=&-\dfrac{1}{4}\int^{a}_{1}\left\{\dfrac{x}{(x^2+b^2)^2}-\dfrac{x}{(x^2+1)^2}\right\}dx \\
=&-\dfrac{1}{4}\left[-\dfrac{1}{2}\cdot\dfrac{1}{x^2+b^2}-\left(-\dfrac{1}{2}\right)\cdot\dfrac{1}{x^2+1}\right]^{x=a}_{x=1} \\
=&\dfrac{1}{8}\left[\dfrac{1}{x^2+b^2}-\dfrac{1}{x^2+1}\right]^{x=a}_{x=1} \\
=&\dfrac{1}{8}\left(\dfrac{1}{a^2+b^2}-\dfrac{1}{a^2+1}-\dfrac{1}{1+b^2}+\dfrac{1}{2}\right) \end{align}$$となり、$a \to \infty$、$b \to \infty$ とすると、これは $\dfrac{1}{16}$ に収束する。$a$、$b$のとり方によらず有限確定値に収束するから広義の重積分が定義できて、$$\iint_D \dfrac{xy}{(x^2+y^2)^3}dxdy=\dfrac{1}{16} \ \ \cdots \cdots \text{(答)}$$と求められる。

 

(2)$\displaystyle \iint_D x^2 e^{-(x^2+y^2)}dxdy$ $D:\{(x,y)|\ x,y,\geqq 0\}$

$t=x^2$ と置くと、$$\begin{align} \displaystyle \int^{\infty}_{0} \left|x^2 e^{-x^2}\right|dx &= \int^{\infty}_{0} t e^{-t} \dfrac{1}{2\sqrt{t}}\ dt \\ &=\dfrac{1}{2}\int^{\infty}_{0} e^{-t}t^{\frac{1}{2}}dt \\ &=\dfrac{1}{2}\varGamma\left(\dfrac{3}{2}\right) \\ &=\dfrac{\sqrt{\pi}}{4} \end{align}$$となるので、積分$\displaystyle \int^{\infty}_{0} \left|x^2 e^{-x^2}\right|dx$は存在する。

また、$u=y^2$ と置くと、$$\begin{align} \displaystyle \int^{\infty}_{0} \left|e^{-y^2}\right|dy &= \int^{\infty}_{0} e^{-u} \dfrac{1}{2\sqrt{u}}\ du \\ &=\dfrac{1}{2}\int^{\infty}_{0} e^{-u}u^{-\frac{1}{2}}du \\ &=\dfrac{1}{2}\varGamma\left(\dfrac{1}{2}\right) \\ &=\dfrac{\sqrt{\pi}}{2} \end{align}$$となるので、積分$\displaystyle \int^{\infty}_{0} \left|e^{-y^2}\right|dy$は存在する。

以上より、$x^2 e^{-(x^2+y^2)}$は$D:\{(x,y)|\ x,y,\geqq 0\}$において積分可能で、$$\begin{align}&\ \ \ \ \ \displaystyle \iint_D x^2 e^{-(x^2+y^2)}dxdy \\ &=\left(\int^{\infty}_{0} x^2 e^{-x^2}dx\right)\left(\int^{\infty}_{0} e^{-y^2}dy\right) \\ &=\dfrac{\sqrt{\pi}}{4}\cdot\dfrac{\sqrt{\pi}}{2} \\ &=\dfrac{\pi}{8} \ \ \cdots \cdots \text{(答)}\end{align}$$と求められる。

 


 

復習例題は設定していません。

 


前に戻る トップへ戻る